System Status Check for AYJR

These checks ensure that you can attempt the paper in a smooth manner.

If everything is correct, you should be able to see the question loading properly below along with a diagram:

Sample Question Preview

Let $m$ and $n$ be the coefficients of seventh and thirteenth terms respectively in the expansion of $\left(\frac{1}{3} x^{\frac{1}{3}}+\frac{1}{2 x^{\frac{2}{3}}}\right)^{18}$. Then $\left(\frac{\mathrm{n}}{\mathrm{m}}\right)^{\frac{1}{3}}$ is :

Sample Diagram Preview